Đến nội dung

YoLo nội dung

Có 216 mục bởi YoLo (Tìm giới hạn từ 28-04-2020)



Sắp theo                Sắp xếp  

#717225 Mong mọi người cho em kinh nghiệm học tốt Toán cao cấp

Đã gửi bởi YoLo on 04-11-2018 - 21:37 trong Kinh nghiệm học toán

:D :D hồi xưa bác trùm cái môn gì mà giờ toàn làm Toán cấp 2 thế.

Chắc bác này "chùm" Cauchy-Schwarz đấy anh

Uii thế à, anh giỏi quá, em rất hâm mộ những người giỏi toán, đặc biệt là toán cao cấp. Vậy sau này có bài nào khó em đăng lên, nếu có thể em mong anh sẽ giúp em với ạ.hihi

Em chưa học với đọc nhiều về toán CC nhưng em có lời khuyên cho chị có gì khó thì nên hỏi $2$ bác bangbang  với vutuanhien  chứ hỏi bác kia bác ấy lại tương " Áp dụng bất đẳng thức Cauchy" thì .... :D




#716184 Cho đồ thị G trong không gian 3 chiều

Đã gửi bởi YoLo on 30-09-2018 - 23:56 trong Tổ hợp và rời rạc

Cho đồ thị $G$ xác định trong không gian $3$ chiều gồm $e$ cạnh .Các đỉnh được nối với nhau nếu khoảng cách giữa chúng là $1$. Biết rằng tồn tại $1$ chu trình $Hamilton$ Chứng minh rằng với $e> 1$ thì:

 min $d(v)\leq 1+2(\frac{e}{2})^{0.4}$

Source: Lượm đâu đó trên Aops




#716119 Đề chọn HSG bảng A và chọn đội tuyển tỉnh Hải Phòng

Đã gửi bởi YoLo on 29-09-2018 - 08:30 trong Thi HSG cấp Tỉnh, Thành phố. Olympic 30-4. Đề thi và kiểm tra đội tuyển các cấp.

Còn bài 4: Giả sử có thể thực hiện sau $1$ số hữu hạn bước

Gọi $S_{n}$ là tổng các số trên bảng sau khi thực hiện bước thứ $n$=> $S_{0}=0$

Nếu $n$ lẻ thì $(n-1)^{2}$ chẵn

Khi đó với mọi $i\in \mathbb{N}$ thì $S_{i}\equiv S_{i+1}(mod2)$

Mà sau hữu hạn bước thực hiện được =>$\exists k$ mà $S_{k}=1+2+3+...+n^{2}=\frac{n^{2}(n^{2}+1)}{2}$ là $1$ số lẻ

=> vô lý

Nếu $n$ chẵn mà sau hữu hạn bước có được bảng thỏa mãn bài => khi đó trên bảng có $\frac{n^{2}}{2}$ số chẵn và $\frac{n^{2}}{2}$ số lẻ

Xét hình vuông $(n-2)\times(n-2)$ có tâm trùng với tâm hình vuông $n\times n$

mà $n\geq 7$ => $(n-4)^{2}>8\Rightarrow (n-2)^{2}>\frac{n^{2}}{2}$

=> trong hình vuông đó $\exists 2$ ô khác tính chẵn lẻ

Ban đầu $2$ ô đó có giá trị $0$ mà khi thực hiện các bước như trên dễ thấy tính chẵn lẻ của hiệu $2$ số này không đổi => chúng ko thể $\neq$ tính chẵn lẻ

=> GIả sử sai




#716113 Kì thi chọn đội tuyển quốc gia năm học 2018-2019 tỉnh Lâm Đồng

Đã gửi bởi YoLo on 28-09-2018 - 22:58 trong Thi HSG cấp Tỉnh, Thành phố. Olympic 30-4. Đề thi và kiểm tra đội tuyển các cấp.

Bài 6:

Phân hoạch các điểm thành các tập sau cho dễ nhìn

$A_{i}=(2018(i-1)+1;2018(i-1)+2;...;2018i)$ với $i=1,2018$

Khi gọi $B_{i}$ tập con các phần tử có dạng $2018(i-1)+k$ với $k=1,2014$ của tập $A_{i}$ phải liên kết với phần tử có dạng tương ứng của tập $B_{i+1};B_{i+2};B_{i+3}$ (Coi như chỉ số $i$ là nhỏ nhất)

Khi đó số tứ giác tối đa được tạo thành từ các điểm trên là $\left \lfloor \frac{2018}{4} \right \rfloor.2014=1015056$ tứ giác

Xét các tập $C_{i}=A_{i}\setminus B_{i}$ với $i=1,2018$

Khi đó xét bảng ô vuông $4\times 2018$ trong đó trên mỗi dòng thứ $i$ được điền các phần tử theo thứ tự tăng dần của tập $C_{i}$

Khi đó số lượng các tứ giác tối đa có thể tạo thành chính là số cách xếp tối đa các tetromino $(1\times 4)$ trên bảng theo chiều ngang hoặc dọc sao cho không có $2$ tetromino nào đè lên nhau khi đó dễ dàng đếm được số lượng tối đa là $2018$ tứ giác

Vậy tối đa là $1015056+2018=1017074$ tứ giác

Bài 5:

 Thay $x=0$ suy ra $f(y).f(0)=0$

=> $f(0)=0$

thay $y=-1$ suy ra $xf(x)+f(x^{2}).f(-1)=0$

=> $f$ là hàm lẻ

nếu ( dễ thấy nếu $f(1)=0$ thì $f(x)=0$ với mọi $x$

=> $f(x^{2})=\frac{xf(x)}{f(1)}$

thay vào => $xf(x+xy)=xf(x)+\frac{xf(x)}{f(1)}.f(y)$

=> $f(x+xy)=f(x)+\frac{f(x)f(y)}{f(1)}$

thay $y=1$ có $f(2x)=2f(x)$=> $f(2)=2f(1)$

thay $x=y=1$ vào pt ban đầu có $f(2)=f(1)+f(1)^{2}$ => $f(1)=1$

=>$f(x+xy)=f(x)+f(x).f(y)$

thay $y=x-1$ => $f(x^{2})=f(x)+f(x).f(x-1)$ ma $f(x^{2})=xf(x)$

=> $(x-1)f(x)=f(x-1)f(x)$

=> $f(x)=0$ hoặc $f(x)=x$ ( dễ cm ko đồng thời xảy ra cả $2$)




#716064 $\exists i,j (1 \le i < j \le n)$ để $...

Đã gửi bởi YoLo on 27-09-2018 - 20:26 trong Dãy số - Giới hạn

$\Rightarrow \left | x_1+x_2+...+x_n \right |=\left | x_m+\sum _{1\leqslant k\leqslant n,k\neq m}x_k \right |\geqslant \left | \left | x_m \right |-\left | \sum _{1\leqslant k\leqslant n,k\neq m}x_k \right | \right |\geqslant | a_n-(a_1+a_2+...+a_{n-1}) |$

Anh xem lại hộ e đoạn này

có $\left | \sum_{1\leq k\leq n,k\neq m}x_{k}\right |\leq a_{1}+a_{2}+...+a_{n-1}\Rightarrow \left | x_{m} \right |-\left | \sum_{1\leq k\leq n,k\neq m}x_{k}\right |\geq a_{n}-(a_{1}+a_{2}+...+a_{n-1})$

Khi đó nếu kết quả của vế trái $< 0$ thì khi trùm dấu trị tuyệt đối vào thì BĐT đổi chiều

có  $x_{m}=-(\sum_{1\leq k\leq n,k\neq m}x_{k})\Rightarrow \left | x_{m} \right |=\left | \sum_{1\leq k\leq n,k\neq m}x_{k}\right | \Rightarrow 0\geq a_{n}-(a_{1}+a_{2}+...+a_{n})$

chả suy ra đc điều vô lý gì trong cm của anh ở phần trên

Vế $1$ =$0$ vế $3=0$ vậy anh đã cm $0\geq 0$




#714612 Tìm số phần tử lớn nhất của tập đặc biệt M.

Đã gửi bởi YoLo on 20-08-2018 - 22:53 trong Tổ hợp - Xác suất và thống kê - Số phức

(RMN_MO 201Bài 3 : Cho tập hợp khác rỗng $S\subset \mathbb{Z}$ thoả mãn :

(i) Tồn tại 2 phần tử $a,b\in S$ : $(a,b)=(a-2,b-2)=1$.

(ii) Nếu $x,y\in S$ thì $x^2-y\in S$ (x,y không nhất thiết phân biệt ).

Chứng minh rằng $S=\mathbb{Z}$

$

Cm $1\in S$

=> $1^{2}-1=0\in S$

=> $0^{2}-1=-1\in S$

=> $1^{2}-(-1)=2\in S$

=> $0^{2}-2=-2\in S$

.......

Làm liên tiếp nhiều lần như vậy tạo ra được tất cả các số vét hết tập $\mathbb{Z}$




#714212 Có tất cả bao nhiêu số tự nhiên thoả mãn đề bài

Đã gửi bởi YoLo on 11-08-2018 - 22:18 trong Tổ hợp và rời rạc

(Đáp án này nhỏ hơn đáp án của bạn YoLo ở trên rất nhiều)

A cho e hỏi lời giải của e mắc chỗ nào e suy đi suy lại vẫn ra vậy

mà e đã thử với TH có tối đa là $2$ chữ số và $3$ chữ số và thấy KQ ko có j sai




#714158 Có tất cả bao nhiêu số tự nhiên thoả mãn đề bài

Đã gửi bởi YoLo on 10-08-2018 - 17:58 trong Tổ hợp và rời rạc

Hỏi có tất cả bao nhiêu số tự nhiên chia hết cho $9$ mà mỗi số có tối đa $2008$ chữ số và trong đó có ít nhất hai chữ số $9$

Nếu là số có $1$ chữ số chỉ có $0;9$

Tổng quát xét số có $k$ chữ số ( $k\geq 2$)

Có  $10$ cách chọn chữ số hàng đơn vị ( xõa từ $0$ đến $9$)

      $10$ cách chọn chữ số hàng chục

        ..........

      $10$ cách chọn chữ số hàng thứ $k-1$

Tổng các chữ số các hàng đơn vị , chục, ...., $k-1$  có thể $\equiv 0,1,2,3,4,5,6,7,8(mod9)$

Vì là chữ số hàng thứ $k$ ( không thể bằng $0$) nên $\exists !1$ cách chọn chữ số hàng thứ $k$)

nên với số có $k$ chữ số  thì có $10^{k-1}$ số tm

vậy tất cả $\sum_{k=2}^{2008}10^{k-1}+2$




#714114 Đề thi HSG toán 10 chuyên KHTN học 2018-2019

Đã gửi bởi YoLo on 09-08-2018 - 23:22 trong Thi HSG cấp Tỉnh, Thành phố. Olympic 30-4. Đề thi và kiểm tra đội tuyển các cấp.

Câu V. CHo $a,b$ là hai số nguyên dương nguyên tố cùng nhau. Xét lưới điểm nguyên trên mặt phẳng tọa độ. Có hai loại bước chuyển, một bước chuyển lọa $A$ là di chuyển từ điểm $(x,y)$ đến một trong $4$ điểm sau: $(x \pm a, y\pm a)$, bước chuyển lọa $B$ di chuyển từ điểm $(x,y)$ đến một trong $4$ điểm sau: $(x \pm b, y \pm b)$. Giả sử ban đầu ta ở vị trị $(0,0)$ ta thực hiện luân phiên các bước chuyển loại $A$ và $B$, bắt đầu từ loại $A$ trước. Hỏi sau một số hữu hạn bước ta có thể đến được những điểm nào trong mặt phẳng. 

Mk xin phép  được chém gió

Bổ đề : Cho $2$ số nguyên dương $a,b$ nguyên tố cùng nhau khi đó $\exists m,n\in \mathbb{Z}$ sao cho $ma+nb=1$

Ở đây mở rộng ra chút nữa là$\exists m,n\in \mathbb{Z}$ sao cho $ma+nb=1$

Cách chứng minh sử dụng thuật toán Euclid

Lời giải:

Dễ thấy khi thực hiện như trên thì hoành độ và tung độ của mỗi điểm được tạo thành có dạng $ma+nb$ với $m,n\in \mathbb{Z}$

Áp dụng bổ đề , kể cả khi thực hiện liên tiếp $2$ loại bước chuyển như trên thì luôn $\exists m,n\in \mathbb{Z}$ sao cho $ma+nb=1$ và $\exists m,n\in \mathbb{Z}$ sao cho $ma+nb=-1$

Khi đó sau một số hữu hạn thực hiện thay phiên các bước chuyển thì từ $1$ điểm $(x;y)$ bất kì có thể tạo ra được các điểm $(x+1;y+1);(x+1;y-1);(x-1;y+1);(x-1;y-1)$

Ban đầu cho điểm $(0;0)$ nên sau khi thực hiện các bước chuyển có thể đến được tất cả các điểm $(x;y)$ $x,y\in \mathbb{Z}$ thỏa mãn $x+y\vdots 2$

Ngược lại giả sử có thể đến đc điểm $(x;y)$ sao cho $x+y$ lẻ

dễ thấy $(x\pm a)+(y\pm a)\equiv x+y(mod 2)$

hay $(x\pm b)+(y\pm b)\equiv x+y(mod 2)$

mà bắt đầu $(0;0)$  => các điểm sau phải có tổng tung + hoành chia hết cho $2$

Vậy  có thể đến được tất cả các điểm $(x;y)$ $x,y\in \mathbb{Z}$ thỏa mãn $x+y\vdots 2$




#714093 cho đa thức P(x) bậc n có n nghiệm dương phân biệt.

Đã gửi bởi YoLo on 09-08-2018 - 19:52 trong Đa thức

cho đa thức P(x) bậc n có n nghiệm dương phân biệt. Chứng minh rằng đa thức G(x)=P(x)-P'(x) cũng có n nghiệm dương phân biệt

$P(x)$ có $n$ nghiệm dương phân biệt gọi là $a_{n}>a_{n-1}>...>a_{2}>a_{1}>0$

Khi đó $P(x)=m(x-a_{1})(x-a_{2})(x-a_{3})...(x-a_{n})$ ( không mất tính TQ giả sử $m>0$ )

Khi ấy $P'(x)=$m$\sum_{j=1}^{n}\prod_{i=1;i\neq j}^{n}(x-a_{i})$

Xét $2$ TH : $n$ chẵn và $n$ lẻ

Nếu $n$ chẵn ta thấy $P(a_{1})=P(a_{2})=P(a_{3})=...=P(a_{n})=0$

cũng đồng thời suy ra được $P'(a_{1})<0;P'(a_{2})>0;P'(a_{3})<0;P'(a_{4})>0;...;P'(a_{n})>0$

suy ra $G(a_{1})>0;G(a_{2})<0;G(a_{3})>0;G(a_{4})<0;...;G(a_{n})<0$

Vì $P(x)$ là đa thức có bậc $n$  => $P'(x)$ là đa thức có bậc không quá $n-1$

=> $G(x)$ là đa thức có bậc $n$ với hệ số cao nhất là $m>0$

=> $\lim_{x\rightarrow +\propto }G(x)=+\propto$ => $\exists a>a_{n}$ mà $G(a)>0$

Theo định lý về giá trị trung gian

=> $G(x)$ có  $n$ nghiệm dương phân biệt lần lượt thuộc vào các khoảng $(a_{1};a_{2});(a_{2};a_{3});(a_{3};a_{4});...;(a_{n};a)$

$G(x)$ có bậc $n$

=> $G(x)$ có đúng $n$ nghiệm dương phân biệt

TH $n$ lẻ hoàn toàn tương tự chỉ việc đổi tất cả $>0$ thành $<0$ và ngược lại

P/s: Bài này cũng có thể dùng Rolle nhưng cũng tương tự




#713844 Trong không gian cho $2n$ điểm phân biệt ($n>4$),...

Đã gửi bởi YoLo on 05-08-2018 - 08:06 trong Tổ hợp - Xác suất và thống kê - Số phức

Trong không gian cho $2n$ điểm phân biệt ($n>4$), $n \in \mathbb{N}$, trong đó không có 3 điểm nào thẳng hàng và trong 2n điểm đó có đúng n điểm cùng nằm trên 1 mặt phẳng. Tìm n sao cho từ 2n điểm đã cho tạo ra đúng 505 mặt phẳng phân biệt

Dễ thấy cứ $3$ điểm bất kì tạo thánh $1$ mặt phẳng

Chia $2n$ điểm trên thành $2$ nhóm

Nhóm $1$ :$n$ điểm cùng thuộc cùng $1$ mặt phẳng

Nhóm $2$: $n$ điểm còn lại nghĩa là trong $n$ điểm này không có $4$ điểm nào cùng $\in$ $1$ nửa mặt phẳng

Vì có đúng $n$ điểm cùng nằm trên $1$ mặt phẳng ( nên nếu chọn ra $3$ điểm từ $n$ điểm đó thì sẽ trùng với mặt phẳng mà $n$ điểm đó cùng thuộc nên sẽ chỉ tính là $1$ mặt phẳng

Như vậy số mặt phẳng được tạo ra từ $3$ điểm bao gồm

+) $1$ điểm thuộc nhóm $1$ và $2$ điểm thuộc nhóm $2$

+) $2$ điểm thuộc nhóm $1$ và $3$ điểm thuộc nhóm $2$

+) $3$ điểm thuộc nhóm $2$

Vậy tổng số mặt phẳng được tạo thành là $\binom{n}{1}.\binom{n}{2}+\binom{n}{2}.\binom{n}{1}+\binom{n}{3}+1$

Thay vào giải phương trình




#713296 $f(x+f(y))=y^2+g(x)$

Đã gửi bởi YoLo on 26-07-2018 - 18:22 trong Phương trình hàm

Có tồn tại hay không f(x), g(x)
sao cho $ f: R \to R;$ $g: R \to R; $ và $f(x+f(y))=y^2+g(x)$

Bài này được đăng trong mục "Thách thức toán học" , tạp chí Pi tháng 6 và hiện chưa có đáp án, bạn không nên đăng để đảm bảo tính công bằng cho người giải bài




#713007 Định lý Beatty

Đã gửi bởi YoLo on 22-07-2018 - 10:28 trong Chuyên đề toán THPT

Ai có thể cho mình xin $1$ chứng minh về định lý Beatty

Trước đó mình xin phát biểu lại định lý

Phát biểu

Cho $a;b$ là các số vô tỷ dương sao cho $\frac{1}{a}+\frac{1}{b}=1$

Khi đó $2$ dãy vô hạn

\left \{ A_{n} \right \}=\left \{ \left \lfloor a \right \rfloor \right ,\left \lfloor 2a \right \rfloor,\left \lfloor 3a \right \rfloor,...,\left \lfloor na \right \rfloor\}

\left \{ B_{n} \right \}=\left \{ \left \lfloor b \right \rfloor \right ,\left \lfloor 2b \right \rfloor,\left \lfloor 3b \right \rfloor,...,\left \lfloor nb \right \rfloor\}

Lập thành $2$ phân hoạch của tập số nguyên dương $\mathbb{N}^{*}$

P/s: Latex máy mình chả hiểu sao copy ra nó ghi lỗi xử lý toán




#713006 1+1=''Quả cam" ?

Đã gửi bởi YoLo on 22-07-2018 - 09:58 trong Toán học lý thú

Đồ điên

P/s: Bài viết chỉ mang tính giải trí mà, bạn không cần gắt vậy đâu :icon6:

Và đây là $1$ chứng minh nho nhỏ cho sự "xờ làm" của mình các bạn có thể đưa thêm cm khác

Giải

Xét tập số nguyên dương $\mathbb{Z}^{+}$ và tập hợp các trái cam $T$

Khi đó tích Descarte của tập $\mathbb{Z}^{+}\times \mathbb{Z}^{+}$ là phần tử có dạng $(a;b)$

Xét ánh xạ $f:\mathbb{Z}^{+}\times \mathbb{Z}^{+}\rightarrow T$

dễ thấy nếu $a\in \mathbb{Z}^{+};b\in \mathbb{Z}^{+}\Rightarrow a+b\in \mathbb{Z}^{+}$

Ký hiệu $f(a;b)=a+b$

mà $f(a;b)=$ "Trái cam" ( vì ảnh của $(a;b)$ thuộc $T$ )

cho $a=1;b=1$ khi đó $1+1=$ "Trái cam"




#712975 1+1=''Quả cam" ?

Đã gửi bởi YoLo on 21-07-2018 - 21:49 trong Toán học lý thú

Hãy sử dụng các kiến thức đã học qua để cmr

1+1= "Quả cam"

P/s: Nghiêm cấm tuyên truyền dưới mọi hình thức tới trẻ em dưới 6 tuổi và trẻ em đang tính bằng que tính :icon6:




#712852 (1985)!+(1986)! chia hết cho 1987

Đã gửi bởi YoLo on 19-07-2018 - 23:31 trong Số học

Mình nghĩ bạn chỉ cần ký hiệu "!" thay cho giai thừa thôi :))

 

$(1985)!+(1986)! =(1985)!+(1985)!.1986= (1985)!.1987 \vdots 1987$ 

(ĐPCM)

Hai dấu thang của bạn này đưa ra là tích của các số chẵn hoặc lẻ mà ko phải giai thừa bình thường đâu

 

 

Kí hiệu: 1.3.5...(2n-1)=(2n-1)!!
              2.4.6...(2n)=(2n)!!
CMR: (1985)!!+(1986)!! chia hết cho 1987

 

Giải đơn giản thôi

$1\equiv -1986(mod1987)$

$3\equiv -1984(mod 1987)$

...........

$1985\equiv -2(mod 1987)$

Nhân theo vế suy ra

$1.3.5...1985\equiv -2.4.6...1986(mod1987)$

suy ra đpcm




#712108 Tìm hàm f:R R

Đã gửi bởi YoLo on 07-07-2018 - 20:37 trong Phương trình hàm

Tìm hàm $f:\mathbb{R}\rightarrow \mathbb{R}$ thỏa mãn

1.$f(x+f(y+f(x)))=f(x+y)+x$

P/s: Đây là $2$ câu riêng biệt mk chế ra ko biết có bị trùng ở đâu ko :icon6:




#711881 Marathon tổ hợp rời rạc VMF 2018

Đã gửi bởi YoLo on 03-07-2018 - 00:24 trong Tổ hợp và rời rạc

Chỉ cần tồn tại chứ không cần đúng với mọi cách chọn các số 1 và -1. Tức là nói theo cách khác; khi xếp trên đường tròn; nếu có một điểm mà tổng thành phần gồm các số liên tiếp nhau bắt đầu từ điểm đó theo mọi phía là một số dương thì số lớn nhất các số -1 là bao nhiêu. Mình nghĩ bạn hiểu sai đề ở chỗ nào đó; hoặc là mình không hiểu lời giải của bạn

Mình lại hiểu đề bài là với max số lượng các số  $-1$ đó thì với mọi cách sắp xếp phải thỏa mãn bài

Mình xin giải lại như sau

cho xếp $2017$ số trên vòng tròn theo chiều kim đồng hồ

theo giả thiết thì $\exists 1$ số trên đường tròn mà tổng $k$ số về 2 phía của số đó lớn hơn $0$

Giả thiết cho đúng với mọi $k$ nên với $k=1008$ thì tổng $1009$ số về $2$ phía của số $a_{i}>0$

với $k=1$ => $a_{i}+a_{i-1}>0;a_{i}+a_{i+1}>0$

=> $a_{i}=a_{i-1}=a_{i+1}=1$

Xét $k=1008$ => $1009$ số theo chiều kim đồng hồ có tổng $>0$=> phải có tối thiểu $505$ số $1$

                             $1009$ số theo ngược kim đồng hồ có tổng $>0$=> phải có tối thiểu $505$ số $1$

=> có tối thiểu là $1010$ số $1$

=> có tối đa $1007$ số $-1$

nếu số lượng $-1$ là $1007$ ta có $1;1;-1;1;-1;1;...;-1;1$ xếp theo chiều kim đồng hồ khi đó số $a_{1}$ là số mà tổng $k$ số bất kì về 2 phía nó  $>0$

=> số lượng lớn nhất các số $-1$ là $1007$




#711637 CMR : Tồn tại một đường tròn đi qua 3 điểm trong số chúng mà không chứa các đ...

Đã gửi bởi YoLo on 26-06-2018 - 23:41 trong Toán rời rạc

Trên mặt phẳng cho n điểm ( $n \geq 3$), trong đó không có 3 điểm nào thẳng hàng.CMR: Tồn tại 1 đường tròn đi qua 3 điểm trong số các điểm đã cho mà không chứa các trong nó điểm nào trong số các điểm còn lại.

Đề bài thiếu : không có $4$ điểm nào cùng thuộc $1$ đường tròn ( nhỡ $n$ điểm này cùng thuộc $1$ đường tròn)

Có $n$ điểm mà ko có $3$ điểm nào thẳng hàng luôn tồn tại $2$ điểm sao cho $n-2$ điểm còn lại $\in$ cùng một nửa mặt phẳng có bờ là đường thẳng chứa đoạn thẳng có $2$ mút là $2$ điểm trên

gọi $2$ điểm đó là $A_{1},A_{2}$ và $n-2$ điểm còn lại là $B_{1},B_{2},B_{3},...,B_{n-2}$

Xét các góc $\widehat{A_{1}B_{i}A_{2}} (i=1,2,3,..,n-2)$

luôn tồn tại một góc có số đo lớn hơn hẳn những góc còn lại giả sử là $\widehat{A_{1}B_{m}A_{2}}$

khi đó vẽ đường tròn ngoại tiếp TG này

Dễ cm nếu $\exists 1$ điểm nằm trong đường tròn đó gs là $B_{n}$ thì $\widehat{A_{1}B_{n}A_{2}}>\widehat{A_{1}B_{m}A_{2}}$

=> vô lý vì góc trên là lớn nhất

P/s : Bài náy có thể mở rộng là có thể vẽ $1$ đường tròn chứa đúng $m$ điểm với ($m\leq n$)




#711148 Chứng minh rẳng tồn tại vô hạn các số nguyên dương n thoả mãn : $ n^...

Đã gửi bởi YoLo on 17-06-2018 - 22:53 trong Số học

Quan trọng là có "chỉ ra được $2$ giá trị nguyên dương khác $1$ của $n$ thỏa mãn đề bài" hay không ~O)

Nếu như anh nói chả nhẽ đề sai, , vậy hỏi người ra đề ???  
A xem e chứng minh có nhầm chỗ nào ko




#711116 Chứng minh rẳng tồn tại vô hạn các số nguyên dương n thoả mãn : $ n^...

Đã gửi bởi YoLo on 16-06-2018 - 23:05 trong Số học

Chứng minh rẳng tồn tại vô hạn các số nguyên dương n thoả mãn :  $ n^2 \mid 3^{n} + 1 $

Giả sử tồn tại hữu hạn các số nguyên dương  $p_{1},p_{2},p_{3},...,p_{k}$ thỏa mãn bài và $p_{k}>p_{k-1}>...>p_{1}>1$

Chỉ ra $2$ giá trị nguyên dương khác $1$ bất kỳ của $n$ thỏa mãn bài => $k\geq 2$

Nếu có số nguyên dương $n$ chẵn thỏa mãn đề bài khi đó $3^{n}+1\equiv 2(mod4)$ , $n^{2}\equiv 0(mod4)$ (vô lý)

=> tất cả các ước số nguyên tố của $p_{1},p_{2},p_{3},...,p_{k}$ đều là số lẻ

Đặt $a=LCM(p_{k},p_{k-1})$=> $a$ lẻ

=> $a$ lớn hơn tất cả các số $p_{i}$ bên trên 

mặt khác $\frac{a}{p_{k}},\frac{a}{p_{k-1}}$ đều là các số lẻ

=>$3^{a}+1\vdots 3^{p_{k}}+1\vdots p_{k}^{2}$

và $3^{a}+1\vdots 3^{p_{k-1}}+1\vdots p_{k-1}^{2}$

=> $3^{a}+1\vdots LCM(p_{k}^{2},p_{k-1}^{2})$

=> $3^{a}+1\vdots a^{2}$

=> $a$ là một số lớn hơn tất cả số kia tm bài

=> gs sai

=> có vô hạn $n$ thỏa mãn

P/s : Hình như có thể thay số $3$ bằng bất kỳ số nguyên dương lẻ khác $1$




#710933 ĐỀ THI TUYỂN SINH VÀO LỚP 10 CHUYÊN TIN HÀ NỘI NĂM HỌC 2018-2019

Đã gửi bởi YoLo on 14-06-2018 - 22:00 trong Tài liệu - Đề thi

Bài V:

 Không mất tính tổng quát giả sử $2018$ đỉnh đó là đỉnh của $1$ đa giác đều nội tiếp đường tròn

An đi trước vẽ đường kính của đường tròn đó mà đi qua $2$ trong số $2018$ điểm trên

Khi đó đường tròn chia làm $2$ nửa đến lượt Bình. Bình sẽ vẽ 1 dây cung bất kỳ nối bởi 2 điểm . dây đó không thể cắt đường kính mà An vừa vẽ

=> mọi dây Bình vẽ phải nằm về 1 phía của đường tròn

 Khi đó đến lượt An sẽ vẽ dây cung đối xứng với dây cung BÌnh vẽ qua đường kính ban đầu trên nửa đường tròn còn lại

Tiếp theo đến lượt Bình vẽ dây cung tùy ý trên một trong 2 nửa đường tròn chia bởi đường kính, An lại tiếp tục lấy đối xứng

Cứ thực hiện như vậy đến $1$ lúc nào đó Bình sẽ thua, giả sử An thua , tức là sau khi Bình vẽ dây cung $AB$ nào đó An sẽ không thể vẽ được dây nào nữa

nghĩa là dây cung đối xứng với dây Bình vừa vẽ gọi là $A'B'$ sẽ cắt 1 dây cung từ trước đó gọi là dây $CD$ mà $\exists C'D'$ là dây đối xứng với dây $CD$ qua đường kính được vẽ từ trước mà $A'B'$ cắt $CD$ suy ra $AB$ cắt $C'D'$ suy ra dây Bình vẽ đã cắt 1 dây vẽ từ trước => vô lý

Vậy Bình thua

P/s: Bài tổ đề Tin còn hay hơn đề Toán




#710536 Đề thi Chọn đội tuyển KHTN (vòng 1)

Đã gửi bởi YoLo on 10-06-2018 - 23:42 trong Thi HSG cấp Tỉnh, Thành phố. Olympic 30-4. Đề thi và kiểm tra đội tuyển các cấp.

Bài 4 vẫn chưa có lời giải ?

Không biết mình làm thử đứng hay sai mn góp ý

Theo quy nạp $f(x+n)=f(x)+n\forall x$ n nguyên

có $f(x+x^{2}-x)=f(x)+x^{2}-x\forall x$ nguyên

=> $f(x^{2})=f(x)+x^{2}-x\forall x$ nguyên

=> $f(x)^{2}-f(x)=x^{2}-x$ nguyên

=> $f(x)=x\forall x$ nguyên  hoặc $f(x)=1-x\forall x$ nguyên ($f(x)=$ cả $2$ thì dễ dàng cm vô lý

$f(x)=1-x\forall x$ nguyên loại

=>$f(x)=x\forall x$ nguyên




#710410 $(x+1)^n+x^n+1 \vdots x^2+x+1$

Đã gửi bởi YoLo on 09-06-2018 - 23:14 trong Đa thức

2. Tìm n sao cho đa thức $(x+1)^n+x^n+1$ chia hết cho $x^2+x+1$.

Chắc $n$ nguyên dương nhỉ

Có $f(x)=(x+1)^{n}+x^{n}+1=(x^{2}+x+1).Q(x)$

Có $x^{2}+x+1=0$ có 2 nghiệm phức là $x_{1}=\frac{-1}{2}+\frac{\sqrt{3}}{2}i;x_{2}=\frac{-1}{2}-\frac{\sqrt{3}}{2}i$

suy ra $f(x_{1})=0;f(x_{2})=0$

=> $(\frac{1}{2}+\frac{\sqrt{3}}{2}i)^{n}+(\frac{-1}{2}+\frac{\sqrt{3}}{2}i)^{n}+1=0$

và $(\frac{1}{2}-\frac{\sqrt{3}}{2}i)^{n}+(\frac{-1}{2}-\frac{\sqrt{3}}{2}i)^{n}+1=0$

từ 2 cái này ta có thể dễ dàng suy ra $n$ chẵn (phản chứng $n$ lẻ thì vô lý)

khi $n$ chẵn viết lại được thành $x_{1}^{n}+x_{2}^{n}+1=0$

mà theo Vi-ét $x_{1}+x_{2}=-1;x_{1}.x_{2}=1$

=> $n=2$




#710401 $(x+1)^n+x^n+1 \vdots x^2+x+1$

Đã gửi bởi YoLo on 09-06-2018 - 22:25 trong Đa thức

3. Cho $m,n \in \mathbb{Z}$ và $m,n \ge 2$. CMR các đa thức:

$$ f(x)=1+x+x^2+...+x^{m-1}$$

$$ g(x)=1+x+x^2+...+x^{n-1}$$

là nguyên tố cùng nhau khi và chỉ khi $m,n$ là hai số nguyên tố cùng nhau.

Ta viết lại $f(x)=\frac{x^{m}-1}{x-1};g(x)=\frac{x^{n}-1}{x-1}$

Vậy ta chứng minh $(m,n)=1\Leftrightarrow (x^{m}-1,x^{n}-1)=x-1$

Gọi $D(x)$ là ước chung lớn nhất của $x^{m}-1$ và $x^{n}-1$

Có $(m,n)=1$ khi đó $\exists u,v$ là các số nguyên dương sao cho $mu-nv=1$ hay $mv-nu=1$

Xét 1 TH : TH kia hoàn toàn tương tự

có $x^{m}-1\vdots D(x)\Rightarrow x^{mu}-1\vdots D(x)\Rightarrow x^{nv+1}-1\vdots D(x)\Rightarrow x(x^{nv}-1)+x-1\vdots D(x)$ (vì $mu=nv+1$)

mà $x^{n}-1\vdots D(x)\Rightarrow x^{nv}-1\vdots D(x)\Rightarrow x-1\vdots D(x)\Rightarrow D(x)=x-1$

=> đpcm

Chiều ngược lại đơn giản rồi $(m,n)=d\Rightarrow (x^{m}-1;x^{n}-1)=x^{d}-1\Rightarrow d=1$

P/s: Dấu $\vdots$ ở đây mk dùng hơi bừa bãi chỉ là cho nhanh thôi chứ đánh ra "chia hết cho" lâu lắm